Mental Health 1 exam practice questions – Flashcards

Unlock all answers in this set

Unlock answers
question
1. J has been admitted to the psychiatric hospital for assessment and evaluation. What behavior might indicate that J has a mental illness? a. She is able to see the difference between the "as if" and the "for real." b. She describes her mood as consistently sad, discouraged, down in the dumps, hopeless. c. She responds to the rules, routines, and customs of any group to which she belongs. d. Within the limits set by her abilities, she can perform tasks she attempts.
answer
b
question
2. An outcome for S is that she will demonstrate mentally healthy behavior. Which behavior indicates the outcome is being met? a. She is aggressive in meeting her needs without considering the rights of others. b. She behaves without considering the consequences of her actions. c. She sees herself as approaching her ideals, and as capable of meeting demands. d. She passively allows others to assume responsibility for major areas of her life.
answer
c
question
3. Strength of the multiaxial DSM-IV is that it: a. includes nursing as well as medical diagnoses b. assessments of several aspects of functioning are made c. it employs the framework of a specific theory d. plans for treatment and evaluation are included
answer
b
question
4. The nurse must assess the mental health or mental illness of several new clients at the mental health clinic. Some of the traits of mental health include: a. Accurate appraisal of reality b. Ability to work and be productive c. Ability to control one's own behavior d. All of the above
answer
d
question
5. B tells the nurse, "I'm a real freak. I'm a psychiatric patient, in and out of hospitals all the time. None of my friends or relatives is crazy like this." What reply would help B understand the prevalence of mental illness? a. "Comparing yourself with others has no real advantages." b. "Mental illness affects 80% of the adult population in any given year." c. "Nearly 50% of all people age 15 to 55 have had a psychiatric disorder at some time in their lives." d. Unfortunately, there are no answers to your question.
answer
b
question
6. The nurse who interviews K notes she is profoundly depressed with thought of wanting to die. Took an unknown amount of pills yesterday. She has refused to do activities of daily living such as bathing. She also has not eaten for the past 5 days, according to her husband. The nurse will code her global assessment of functioning as a. 100 b. 50 c. 25 d. 10
answer
d
question
7. Maslow offers a theory of human motivation that assumes patients have: a. Developmental tasks and psychosocial crisis b. a hierarchy of needs c. the process of schemata, assimilation and accommodation d. all of the above
answer
b
question
8. Which statement allows the nurse to suspect that the developmental task of infancy, according to Erickson, was not successfully completed? a. "I'm afraid to allow anyone to really get to know me." b "I'm absolutely right, so don't bother saying more." c "I'm so ashamed because I didn't do it correctly in the first place." d "Andy and I are very close friends."
answer
a
question
9. T, a 39-year-old businesswoman and single parent of three, is experiencing many feelings of inadequacy in her job and family situation since her 16-year-old daughter ran away several weeks ago. T seeks the help of a therapist specializing in cognitive therapy. The nurse psychotherapist using cognitive therapy will treat T by a. focusing on unconscious mental processes b. negatively reinforcing an undesirable behavior c. discussing ego states d. helping her identify and change faulty thinking
answer
d
question
10. A therapy that grew out of the need to provide treatment for the posttraumatic stress disorder of returning veterans from WWII and Vietnam is: a. psychoanalysis b. milieu therapy c. systematic desensitization d. short-term dynamic therapy
answer
d
question
11. Which of the principles of planning nursing intervention to meet client goals is violated in this scenario? Goal: Client will, with the aid of staff, remain safe while in the hospital. Interventions: Allow client to attend off unit programming unsupervised. Client can access all personal belongings. a. The interventions are not nursing focused. b. The interventions are not individualized. c. The interventions are not safe. d. The interventions are not relevant to the goal.
answer
c
question
12. The nurse must plan interventions directed toward meeting the client goal: Client will remain safe during hospitalization with the assistance of staff. Which nursing intervention is related to this goal? a. Encourage patient to discuss relationship with family members. b. Assist client to identify three personal weaknesses. c. Observe client for therapeutic effects of psychotropic medication. d. Implement suicide precautions.
answer
d
question
13. A nurse behavior that jeopardizes the boundaries of the nurse-client relationship is a. Focusing on the client's needs. b. Allowing the relationship to become social. c. Suspending judgment. d. Recognizing the need for supervision.
answer
b
question
14. P is being admitted to the psychiatric unit by Nurse G. P was brought to the emergency department after making a suicide attempt by taking an overdose of acetaminophen (Tylenol). P has been lavaged. She appears tense, withdrawn, and frightened. A therapeutic, empathetic response would be that Nurse G tells her: a. "I'd like to sit here with you". b. Tell me more about the suicidal attempt" c. It must be frightening to have just gone through all you have". d. What exactly was going on with you before you tried to hurt yourself".
answer
c
question
Which statement given below would be an example showing that Nurse G is using an open ended exploring question with her patient: a. "Do you think the medications are helping you?" b. "When were you born?" c. "Is there mental illness in your family?" d. "Tell me more about what was going on before your suicide attempt."
answer
d
question
What behavior on the part of Nurse G will produce the evaluation that termination of the therapeutic nurse-client relationship with P, a client, has been handled successfully? a. He gives P his personal telephone number and permission to call after her discharge. b. He avoids upsetting P by gradually focusing on other clients beginning 1 week prior to her discharge. c. He summarizes with P the changes that have happened during their time together and evaluates goal attainment. d. He offers to meet P for coffee and conversation three times a week for 2 weeks after her discharge.
answer
c
question
As Nurse V considers her relationship with K, a client, at what point in the nurse-client relationship should she plan to first address the issue of termination? a. in the working phase b. in the termination phase c. in the orientation phase d. when the client initially brings up the topic
answer
c
question
The nurse caring for an angry manipulative client finds himself feeling angry with the client. The nurse should initially: a. refuse to care for the patient b. let the client know how he feels c. tell the nurse manager to assign the client to another nurse d. deal with his feelings in a supervisory session
answer
d
question
As L converses with the nurse, she states, "I dreamed I was stoned. When I woke up, I was feeling emotionally drained as though I hadn't rested well." If the nurse needs clarification of "stoned," it would be appropriate to say, a. "It sounds as though you were quite uncomfortable with the content of your dream." b. "Can you give me an example of what you mean by stoned?" c. "I understand what you're saying. Bad dreams leave me feeling tired, too." d. "So, all in all, you feel as though you had a rather poor night's sleep?"
answer
b
question
During the first interview with a restless young man, the nurse notices that he does not make eye contact throughout most of the interview. It can correctly be assumed that a. he is not to be trusted in what he says because he is evasive b. he is really feeling sad and can't look the nurse in the eye c. he is shy and the nurse must move slowly d. more data is needed to draw a conclusion
answer
d
question
Which remark by the nurse would be an appropriate way to begin a therapeutic 1:1 session? a. "How shall we start today?" b. "Shall we talk about losing your privileges yesterday?" c. "What happened when your husband came to visit yesterday?" d. Tell me what led up to your hospitalization?
answer
a
question
A client tells the nurse, "I don't think I'll ever get out of here." A therapeutic response would be a. Why do you feel that way? b. "Everyone feels that way sometimes." c. "You don't think you're making progress?" d. "Keep up the good work, and you certainly will."
answer
c
question
In a session J cries as the nurse explores her relationship with her deceased mother. J sobs, "I shouldn't be blubbering like this." A statement by the nurse that will hinder communication is a. "The relationship with your mother is very painful for you." b. "I can see that you feel sad about this situation." c. "Why do you think you are feeling so upset?" d. "Crying is a way of expressing the hurt you're experiencing."
answer
c
question
In a treatment team planning meeting a nurse states her concern about whether the staff is behaving ethically in using restraint to prevent one client from engaging in self-mutilative behavior when the care plan for another self-mutilating client calls for one-on-one supervision. The ethical principle that should govern the situation is a. Beneficence b. Autonomy c. Fidelity d. Justice
answer
d
question
What is the legal significance of the nurse's action when a client verbally refuses his medication and the nurse gives it over his objection? a. The nurse can be charged with battery. b. The nurse can be charged with negligence. c. The nurse can be charged with malpractice. d. No charges can be brought against the nurse.
answer
a
question
S is a newly admitted acutely psychotic client. She is a private client of the chief of staff and a private-pay client. To whom does the psychiatric nurse caring for S owe the duty of care? a. Physician b. Health care agency c. Client d. Profession
answer
c
question
T is an involuntary client on a psychiatric inpatient unit. He asks the nurse for a 5 day release for discharge. The best response for the nurse to make would be a. "I can't give you those forms without your doctor's knowledge." b. "Here is the number for the guardianship and advocacy commission. They will send you the forms." c. "Because you are an involuntary patient you are committed here for 30 days" d. "I will get you the form for you to fill out:
answer
d
question
31.The wife of a client who has delusions of infidelity asks the nurse if there are any circumstances under which the treatment team may violate the client's right to confidentiality. The specialist must reply that confidentiality may be violated a. under no circumstances b. when questions are asked by law enforcement officers c. if the client threatens the life of another person d. at the discretion of the psychiatrist
answer
c
question
Which of the following is a description of the least restrictive alternative? a. Allowing the client to sign a 5 day release for discharge b. Offering the client a by mouth dose of medication instead of an injection c. Signing the petition, cert, and then another cert within 24 hours d. Placing the client in seclusion rather than restraints
answer
b
question
L is experiencing considerable stress. She is in a work environment in which her boss treats her "like a doormat." She states he thinks nothing of demanding that she stay overtime and work on Saturdays whenever he is "in the mood to work." A healthy coping strategy the nurse might suggest for L is a. resigning the position b. starting individual short term therapy c. relying on the support of her family d. employing assertiveness techniques
answer
d
question
The nurse wishes to teach an alternative way of coping to a client experiencing severe anxiety. The nurse will first need to a. use measures designed to lower the client's anxiety b. determine the mode of learning preferred by the client c. devise outcomes and construct a teaching plan d. place the client in seclusion until the anxiety subsides
answer
a
question
A client is noted to have a high level of non-goal-directed motor activity, running from chair to chair in the solarium. He is wide-eyed and seems terror-stricken. He cries, "They're coming! They're coming!" He neither follows staff direction nor responds to verbal efforts to calm him. A nursing diagnosis of high priority is a. Risk for injury b. Self-care deficit c. Disturbed energy field d. Impaired verbal communication
answer
a
question
W is having a series of diagnostic tests. He insists there is nothing wrong with him except a chest cold that he can't "shake off." His wife says he smokes and coughs a lot, has lost 15 pounds, and is easily fatigued. What defense mechanism is W using? a. Regression b. Displacement c. Denial d. Projection
answer
c
question
W has a mass in the left upper lobe of his lung. He is scheduled to undergo a biopsy. When the nurse explains the procedure to him, he seems to have difficulty grasping what she is saying and asks questions such as, "What do you mean I'm going to have surgery? What are they going to do?" His voice is tremulous. His respirations are noticeably rapid at 28 breaths per minute, and his pulse is 110 beats per minute. W can be assessed as having a cognitive problem called a. Rationalization b. Conversion c. Introjection d. selective inattention
answer
d
question
K, a college student who usually gets As, has received a C on a difficult examination. He runs to the college health service. He is disorganized and only partially coherent. In order to reduce K's level of anxiety as quickly as possible, the nurse should first a. stay with him b. tell him that help will come c. question him to discover the events that led to his de-compensation d. tell him he will not be allowed to hurt anyone
answer
a
question
Two staff nurses were considered for promotion. The promotion was announced via a memo on the unit bulletin board. The nurse who was not promoted told another friend, "I knew I'd never get the job. The hospital administrator hates me." If she actually believes this of the administrator, who, in reality, knows little of her, she is demonstrating a. Compensation b. Reaction formation c. Projection d. Denial
answer
c
question
A woman speaking of a rival for her husband's affection says in a gushy, syrupy voice, "She is a lovely person. I simply adore her." The woman may be employing a. Reaction formation b. Projection c. Denial d. Repression
answer
a
question
T states that before taking an examination he feels a heightened sense of awareness and a sense of restlessness. What nursing intervention would be most suitable for assisting T? a. Explain his symptoms as resulting from mild anxiety and discuss the helpful aspects of mild anxiety. b. Advise T to discuss his experience with a physician or psychologist. c. Offer to obtain an order for an anxiolytic that T can take when necessary. d. Listen without comment.
answer
a
question
For the client whose nursing diagnosis is powerlessness related to inability to control compulsive cleaning, the nurse recognizes that the client uses the cleaning to a. temporarily reduce anxiety b. gain a feeling of superiority c. receive praise from friends and family d. ensure the health of household members
answer
a
question
43. Which of the following is considered an obsession in OCD (obsessive compulsive disorder) a. hand washing b. ritualistic counting c. fear of germs d. excessive order
answer
c
question
44. Which of the following is considered a side effect of the anti-anxiety (anxiolytic) medication ativan? a. Decreased tolerance b. Increased sedation c. Increased cognitive function d. All of the above
answer
b
question
45. Which of the following anti-anxiety medications is considered non-addictive? a. valium b. buspirone c. klonopin d. xanax
answer
b
question
1. A new staff nurse completes orientation to a psychiatric unit. This nurse may expect an advanced practice nurse to perform which additional interventions? a. Conduct mental health assessments b. Establish therapeutic relationships c. Individualize nursing care plans d. Prescribe psychotropic medication
answer
d Prescriptive privileges are granted to masters-prepared nurse practitioners who have taken special courses on prescribing medication. The nurse prepared at the basic level is permitted to perform mental health assessments, establish relationships, and provide individualized care planning.
question
Two nursing students discuss their career plans after graduation. One student wants to enter psychiatric nursing. The other asks, "Why would you want to be a psychiatric nurse? The only thing they do is talk. You'll lose all your skills." Select the best response. a. "Psychiatric nurses practice in safer environments than other specialties. Nurse-to-patient ratios must be better because of the nature of the patients' problems." b. "Psychiatric nurses use complex communication skills as well as critical thinking to solve multidimensional problems. I am challenged by those situations." c. "I think I will be good in the mental health field. I did not like clinical rotations in school, so I do not want to continue them after I graduate." d. "Psychiatric nurses do not have to deal with as much pain and suffering as medical-surgical nurses do. That appeals to me."
answer
b The practice of psychiatric nursing requires a different set of skills than medical-surgical nursing, though there is substantial overlap. Psychiatric nurses must be able to help patients with medical as well as mental health problems, reflecting the holistic perspective these nurses must have. Nurse-patient ratios and workloads in psychiatric settings have increased, just like other specialties. Psychiatric nursing involves clinical practice, not just documentation. Psychosocial pain and suffering is as real as physical.
question
A new bill introduced in Congress would reduce funding for care of persons with mental illness. Groups of nurses write letters to their elected representatives in opposition to the legislation. Which role have the nurses fulfilled? a. Advocacy b. Attending c. Recovery d. Evidence-based practice
answer
a An advocate defends or asserts another's cause, particularly when the other person lacks the ability to do that for self. Examples of individual advocacy include helping patients understand their rights or make decisions. On a community scale, advocacy includes political activity, public speaking, and publication in the interest of improving the human condition. Since funding is necessary to deliver quality programming for persons with mental illness, the letter-writing campaign advocates for that cause on behalf of patients who are unable to articulate their own needs.
question
An informal group of patients discusses their perceptions of nursing care. Which comment best indicates a patient perceived the nurse was caring? "My nurse: a. always asks me which type of juice I want to help me swallow my medication." b. explained my treatment plan to me and asked for my ideas about how to make it better." c. told me that if I take all the medicines the doctor prescribes, then I will get discharged soon." d. spends time listening to me talk about my problems. That helps me feel like I'm not alone."
answer
d Caring evidences empathetic understanding as well as competency. It helps change pain and suffering into a shared experience, creating a human connection that alleviates feelings of isolation. The distracters give examples of statements that demonstrate advocacy or giving advice.
question
Which finding best indicates that a patient has a mental illness? The patient: a. responds to rules, routines, and customs of a group. b. reports mood is consistently sad, discouraged, and hopeless. c. performs tasks attempted within the limits set by own abilities. d. is able to see the difference between the "as if" and the "for real."
answer
b The correct response describes a mood alteration, which further reflects mental illness. The distracters describe mentally healthy behaviors.
question
Which finding best indicates that the goal "Demonstrate mentally healthy behavior" was achieved? A patient: a. sees self as approaching ideals and capable of meeting demands. b. seeks others to assume responsibility for major areas of own life. c. behaves without considering the consequences of personal actions. d. aggressively meets own needs without considering the rights of others.
answer
a The correct response describes an adaptive, healthy behavior. The distracters describe maladaptive behaviors.
question
A nurse encounters an unfamiliar psychiatric disorder on a new patient's admission form. To determine criteria used to establish this diagnosis, the nurse should consult which resource? a. Diagnostic and Statistical Manual of Mental Disorders b. A nursing diagnosis handbook c. A psychiatric nursing textbook d. A behavioral health reference manual
answer
a The DSM-IV-TR gives the criteria used to diagnose each mental disorder. The distracters may not contain diagnostic criteria for a psychiatric illness.
question
Which documentation of diagnosis would a nurse expect in a psychiatric treatment setting? a. I Acute renal failure II 75 III Bipolar disorder I, mixed IV Loss of disability benefits 2 months ago V None b. I Schizophrenia, paranoid type II Death of spouse last year III 60 IV None V Diabetes, type 2 c. I Polysubstance dependence II Narcissistic Personality Disorder III 90 IV Hyperlipidemia V Charges pending for assault d. I Major Depression II Avoidant Personality Disorder III Hypertension IV Home destroyed by hurricane last year V 80
answer
d The DSM-IV-TR profiles psychiatric diagnoses on five axes. Each axis defines a specific aspect of the diagnosis. Axis I identifies major clinical disorders. Axis II details personality and developmental disorders. Axis III identifies general medical conditions. Axis IV details psychosocial and environmental problems. Axis V rates the Global Assessment of Functioning.
question
A nurse explains the multiaxial DSM-IV-TR to a psychiatric technician and includes information that it: a. focuses on plans for treatment. b. includes nursing and medical diagnoses. c. classifies problems in multiple areas of functioning. d. uses the framework of a specific biopsychosocial theory.
answer
c The use of five axes requires assessment beyond diagnosis of a mental disorder and includes relevant medical conditions, psychosocial and environmental problems, and global assessment of functioning. The DSM-IV-TR does not include treatment plans or nursing diagnoses. It does not use specific biopsychosocial theories.
question
A nurse wants to find a description of diagnostic criteria for anxiety disorders. Which resource would have the most complete information? a. The ICD-10 b. Nursing Outcomes Classification c. Diagnostic and Statistical Manual of Mental Disorders d. The ANA Psychiatric-Mental Health Nursing Scope and Standards of Practice
answer
c The DSM-IV-TR details the diagnostic criteria for psychiatric clinical conditions. The other references are good resources but do not define the diagnostic criteria.
question
Which comment most clearly shows a speaker views mental illness with stigma? a. "Some mental illnesses are inherited." b. "Most people with mental illness are unmotivated." c. "Severe environmental stress sometimes causes mental illness." d. "Some mental illnesses are brain disorders resulting from changes in how impulses are transmitted."
answer
b Stigma refers to stereotypical, negative beliefs. With respect to mental health and mental illness, stigma often leads to discrimination and uncaring attitudes. Mental illness has multiple causes, including stress, changes in brain structure or function, and genetic transmission.
question
A newly admitted patient is uncommunicative about recent life events. The nurse suspects marital and economic problems, but the social worker's assessment is not yet available. Select the nurse's best action. a. Focus assessment questions on these two topics. b. Ask another patient who shares a room with this patient. c. Avoid seeking information on these topics at this time. d. Refer to axis IV of the DSM-IV-TR in the medical record.
answer
d The admitting physician would use axis IV to record psychosocial and environmental problems pertinent to the patient's situation, providing another source of information for the nurse. Persistent questioning may cause the patient to withdraw. The other distracters demonstrate violation of the patient's privacy rights and are not an effective solution.
question
The Diagnostic and Statistical Manual of Mental Disorders classifies: a. deviant behaviors. b. people with mental disorders. c. present disability or distress. d. mental disorders people have.
answer
d The DSM-IV-TR classifies disorders people have rather than people themselves. The terminology of the tool reflects this distinction by referring to individuals with a disorder rather than as a "schizophrenic" or "alcoholic," for example. Deviant behavior is not generally considered a mental disorder. Present disability or distress is only one aspect of the diagnosis.
question
A nurse participating in a community health fair is asked, "What is the most prevalent mental disorder in the United States?" Select the nurse's best response. a. Schizophrenia b. "Why do you ask?" c. Bipolar disorder d. Alzheimer's disease
answer
d The prevalence for Alzheimer's disease is 10% for persons older than 65 and 50% for persons older than 85. The prevalence of schizophrenia is 1.1% per year. The prevalence of bipolar disorder is 2.6%. It is important for the nurse to provide information rather than probe the reason for the person's question.
question
In the majority culture of the United States, which individual has the greatest risk to be labeled mentally ill? One who: a. describes hearing God's voice speaking. b. is usually pessimistic but strives to meet personal goals. c. is wealthy and gives away $20 bills to needy individuals. d. always has an optimistic viewpoint about life and having own needs met.
answer
a The question asks about risk. Hearing voices is generally associated with mental illness, but in charismatic religious groups, hearing the voice of God or a prophet is a desirable event. Cultural norms vary, which makes it more difficult to make an accurate diagnosis. The individuals described in the other options are less likely to be labeled mentally ill.
question
A patient's relationships are intense and unstable. The patient initially idealizes the significant other and then devalues them, resulting in frequent feelings of emptiness. This patient will benefit from interventions to develop which aspect of mental health? a. Effectiveness in work b. Communication skills c. Productive activities d. Fulfilling relationships
answer
d The information given centers on relationships with others, which are described as intense and unstable. The relationships of mentally healthy individuals are stable, satisfying, and socially integrated. Data are not present to describe work effectiveness, communication skills, or activities.
question
A patient is depressed, mute, and motionless. According to family members, the patient has refused to bathe or eat for a week. The patient's global assessment of functioning score is: a. 100 b. 50 c. 25 d. 10
answer
d The patient is unable to maintain personal hygiene, oral intake, or verbal communication. The patient is a danger to self because of not eating. The distracters represent higher levels of functioning.
question
Which belief will best support a nurse's efforts to provide patient advocacy during a multidisciplinary patient care planning session? a. All mental illnesses are culturally determined. b. Schizophrenia and bipolar disorder are cross-cultural disorders. c. Symptoms of mental disorders are unchanged from culture to culture. d. Assessment findings in mental disorders reflect a person's cultural patterns.
answer
d A nurse who understands that a patient's symptoms are influenced by culture will be able to advocate for the patient to a greater degree than a nurse who believes that culture is of little relevance. The distracters are untrue statements.
question
A nurse is part of a multidisciplinary team working with groups of depressed patients. Half the patients receive supportive interventions and antidepressant medication. The other half receives only medication. The team measures outcomes for each group. Which type of study is evident? a. Prevalence b. Clinical epidemiology c. Descriptive epidemiology d. Experimental epidemiology
answer
b Clinical epidemiology is a broad field that addresses what happens to people with illnesses seen by providers of clinical care. This study is concerned with the effectiveness of various interventions. Prevalence refers to numbers of new cases. Descriptive epidemiology provides estimates of the rates of disorders in a general population and its subgroups. Experimental epidemiology tests presumed assumptions between a risk factor and a disorder.
question
The spouse of a patient with schizophrenia says, "I don't understand how nurturing or toilet training in childhood has anything to do with this incredibly disabling illness." Which response by the nurse will best help the spouse understand this disorder? a. "This illness is the result of genetic factors." b. "Psychological stress is at the root of most mental disorders." c. "It must be frustrating for you that your spouse is sick so much of the time." d. "New findings show that this condition more likely has biological rather than psychological origins."
answer
d Many of the most prevalent and disabling mental disorders have strong biological influences. Genetics are only one part of biological factors. Empathy does not address increasing the spouse's level of knowledge about the cause of the disorder. The other distracters are not established facts.
question
A 40-year-old who lives with parents and works at an unchallenging job says, "I'm as happy as anyone else, even though I don't socialize much outside of work. My work is routine, but when new things come up, my boss explains things a few times to make sure I catch on. At home, my parents make decisions for me, and I go along with their ideas." The nurse should identify interventions to improve this patient's: a. self-concept. b. overall happiness. c. appraisal of reality. d. control over behavior.
answer
a The patient sees self as needing multiple explanations of new tasks at work and allows the parents to make decisions, even though she is 40 years old. These behaviors indicate a poorly developed self-concept.
question
The psychiatric nurse addresses axis I of the DSM as the focus of treatment but must also consider physical health problems that may affect treatment. Which axis contains the desired information? a. II b. III c. IV d. V
answer
b Axis III indicates any relevant general medical conditions. Axis II refers to personality disorders and mental retardation. Together they constitute the classification of abnormal behavior diagnosed in the individual. Axis IV reports psychosocial and environmental problems that may affect the diagnosis, treatment, and prognosis. Axis V is the global assessment of functioning.
question
Select the best response for the nurse who receives a query from another mental health professional seeking to understand the difference between a DSM-IV-TR diagnosis and a nursing diagnosis. a. "There is no functional difference between the two. Both identify human disorders." b. "The DSM-IV-TR diagnosis disregards culture, whereas the nursing diagnosis takes culture into account." c. "The DSM-IV-TR diagnosis is associated with present distress or disability, whereas a nursing diagnosis considers past and present responses to actual mental health problems." d. "The DSM-IV-TR diagnosis affects the choice of medical treatment, whereas the nursing diagnosis offers a framework for identifying interventions for phenomena a patient is experiencing."
answer
d The medical diagnosis is concerned with the patient's disease state, causes, and cures, whereas the nursing diagnosis focuses on the patient's response to stress and possible caring interventions. Both tools consider culture. The DSM-IV-TR is multiaxial. Nursing diagnoses also consider potential problems.
question
A 2-year-old child often displays negative behaviors. The parent says, "My child refuses toilet training and shouts 'No!' when given directions. What do you think is wrong?" Select the nurse's best reply. a. "The child needs firmer control. It is important to set limits now." b. "This is normal for your child's age. The child is striving for independence." c. "There may be developmental problems. Most children are toilet trained by age 2." d. "Some undesirable attitudes are developing. A child psychologist can help you develop a plan."
answer
b This behavior is typical of a child around the age of 2 years whose developmental task is to develop autonomy. The distracters indicate the child's behavior is abnormal.
question
A 26-month-old displays negative behavior, refuses toilet training, and often says, "No!" Which stage of psychosexual development is evident? a. Oral b. Anal c. Phallic d. Genital
answer
b The anal stage occurs from age 1 to 3 years and has as its focus toilet training and learning to delay immediate gratification. The oral stage occurs between birth and 1 year. The phallic stage occurs between 3 and 5 years, and the genital stage occurs between age 13 and 20 years.
question
A 26-month-old displays negative behavior, refuses toilet training, and often says, "No!" Which psychosocial crisis is evident? a. Trust versus mistrust b. Initiative versus guilt c. Industry versus inferiority d. Autonomy versus shame and doubt
answer
d The crisis of autonomy versus shame and doubt relates to the developmental task of gaining control of self and environment, as exemplified by toilet training. This psychosocial crisis occurs during the period of early childhood. Trust versus mistrust is the crisis of the infant. Initiative versus guilt is the crisis of the preschool and early-school-aged child. Industry versus inferiority is the crisis of the 6- to 12-year-old child.
question
A 4-year-old grabs toys from siblings and says, "I want that now!" The siblings cry, and the child's parent becomes upset with the behavior. Using Freudian theory, the nurse can interpret this behavior as a product of impulses originating in the: a. id. b. ego. c. superego. d. preconscious.
answer
a The id operates on the pleasure principle, seeking immediate gratification of impulses. The ego acts as a mediator of behavior and weighs the consequences of the action, perhaps determining that taking the toy is not worth the mother's wrath. The superego would oppose the impulsive behavior as "not nice." The preconscious is a level of awareness.
question
The parent of a 4-year-old rewards and praises the child for helping a younger sibling, being polite, and using good manners. The nurse supports the use of praise related to these behaviors. These qualities will likely be internalized and become part of the child's: a. id. b. ego. c. superego. d. preconscious.
answer
c The superego contains the "thou shalts," or moral standards internalized from interactions with significant others. Praise fosters internalization of desirable behaviors. The id is the center of basic instinctual drives, and the ego is the mediator. The ego is the problem-solving and reality-testing portion of the personality that negotiates solutions with the outside world. The preconscious is a level of awareness from which material can be retrieved easily with conscious effort.
question
A nurse supports a parent for praising a child behaving in a helpful way. When this child behaves with politeness and helpfulness in adulthood, which feeling will most likely result? a. Guilt b. Anxiety c. Humility d. Self-esteem
answer
d The individual will be living up to the ego ideal, which will result in positive feelings about self. The other options are incorrect because each represents a negative feeling.
question
A patient says, "I never know the answers," and "My opinion doesn't count." The nurse correctly assesses that this patient had difficulty resolving which psychosocial crisis? a. Initiative versus guilt b. Trust versus mistrust c. Autonomy versus shame and doubt d. Generativity versus self-absorption
answer
c These statements show severe self-doubt, indicating that the crisis of gaining control over the environment was not successfully met. Unsuccessful resolution of the crisis of initiative versus guilt results in feelings of guilt. Unsuccessful resolution of the crisis of trust versus mistrust results in poor interpersonal relationships and suspicion of others. Unsuccessful resolution of the crisis of generativity versus self-absorption results in self-absorption that limits the ability to grow as a person.
question
Which patient statement would lead the nurse to suspect unsuccessful completion of the developmental task of infancy? a. "I have very warm and close friendships." b. "I'm afraid to allow anyone to really get to know me." c. "I'm always absolutely right, so don't bother saying more." d. "I'm ashamed that I didn't do things correctly in the first place."
answer
b According to Erikson, the developmental task of infancy is the development of trust. The correct response is the only statement clearly showing lack of ability to trust others. Warm, close relationships suggest the developmental task of infancy was successfully completed; rigidity and self-absorption are reflected in the belief one is always right; and shame for past actions suggests failure to resolve the crisis of initiative versus guilt.
question
A patient is suspicious and frequently manipulates others. To which psychosexual stage do these traits relate? a. Oral b. Anal c. Phallic d. Genital
answer
a The behaviors in the stem develop as the result of attitudes formed during the oral stage, when an infant first learns to relate to the environment. Anal-stage traits include stinginess, stubbornness, orderliness, or their opposites. Phallic stage traits include flirtatiousness, pride, vanity, difficulty with authority figures, and difficulties with sexual identity. Genital stage traits include the ability to form satisfying sexual and emotional relationships with members of the opposite sex, emancipation from parents, a strong sense of personal identity, or the opposites of these traits.
question
A patient expresses the desire to be cared for by others and often behaves in a helpless fashion. The patient's needs relate to which stage of psychosexual development? a. Latency b. Phallic c. Anal d. Oral
answer
d Fixation at the oral stage sometimes produces dependent infantile behaviors in adults. Latency fixations often result in difficulty identifying with others and developing social skills, resulting in a sense of inadequacy and inferiority. Phallic fixations result in having difficulty with authority figures and poor sexual identity. Anal fixation sometimes results in retentiveness, rigidity, messiness, destructiveness, and cruelty.
question
A nurse listens to a group of recent retirees. One says, "I volunteer with Meals on Wheels, coach teen sports, and do church visitation." Another laughs and says, "I'm too busy taking care of myself to volunteer to help others." Which developmental task do these statements contrast? a. Trust and mistrust b. Intimacy and isolation c. Industry and inferiority d. Generativity and self-absorption
answer
d Both retirees are in middle adulthood, when the developmental crisis to be resolved is generativity versus self-absorption. One exemplifies generativity; the other embodies self-absorption. This developmental crisis would show a contrast between relating to others in a trusting fashion or being suspicious and lacking trust. Failure to negotiate this developmental crisis would result in a sense of inferiority or difficulty learning and working as opposed to the ability to work competently. Behaviors that would be contrasted would be emotional isolation and the ability to love and commit oneself.
question
Although ego defense mechanisms and security operations are unconsciously determined and designed to relieve anxiety, the major difference is: a. defense mechanisms are intrapsychic and not observable. b. defense mechanisms cause arrested personal development. c. security operations are masterminded by the id and superego. d. security operations address interpersonal relationship activities.
answer
d Sullivan's theory explains that security operations are interpersonal relationship activities designed to relieve anxiety. Because they are interpersonal, they are observable. Defense mechanisms are unconscious and automatic. Repression is entirely intrapsychic, but other mechanisms result in observable behaviors. Frequent, continued use of many defense mechanisms often results in reality distortion and interference with healthy adjustment and emotional development. Occasional use of defense mechanisms is normal and does not markedly interfere with development. Security operations are ego centered.
question
A student nurse says, "I don't need to interact with my patients. I learn what I need to know by observation." An instructor can best interpret the nursing implications of Sullivan's theory to this student by responding: a. "Interactions are required in order to help you develop therapeutic communication skills." b. "Nurses cannot be isolated. We must interact to provide patients with opportunities to practice interpersonal skills." c. "Observing patient interactions will help you formulate priority nursing diagnoses and appropriate interventions." d. "It is important to note patients' behavioral changes, because these signify adjustments in personality."
answer
b Sullivan believed that the nurse's role includes educating patients and assisting them in developing effective interpersonal relationships. Mutuality, respect for the patient, unconditional acceptance, and empathy are cornerstones of Sullivan's theory. The nurse who does not interact with the patient cannot demonstrate these cornerstones. Observations provide only objective data. Priority nursing diagnoses usually cannot be accurately established without subjective data from the patient. The other distracters relate to Maslow and behavioral theory.
question
A psychiatric technician says, "Common sense is the most important part of working with people who have mental illness. Theories are just something to fill up textbooks." The nurse wants to educate the technician by identifying which common use of Sullivan's theory? a. The method nurses use to determine the best sequence for nursing actions b. The ongoing use of restraint and seclusion as behavior-management tools c. The structure of the therapeutic milieu of most behavioral health units d. Assessment tools based on age-appropriate versus arrested behaviors
answer
c The structure of the therapeutic environment has as foci an accepting atmosphere and provision of opportunities for practicing interpersonal skills. Both constructs are directly attributable to Sullivan's theory of interpersonal relationships. Sullivan's interpersonal theory did not specifically consider use of restraint or seclusion. Assessment based on developmental level is more the result of Erikson's theories. Sequencing nursing actions based on patient priority needs is related to Maslow's hierarchy of needs.
question
A nurse uses Maslow's hierarchy of needs to plan care for a patient with mental illness. Which problem will receive priority? The patient: a. refuses to eat or bathe. b. reports feelings of alienation from family. c. is reluctant to participate in unit social activities. d. is unaware of medication action and side effects.
answer
a The need for food and hygiene are physiological and therefore take priority over psychological or meta-needs in care planning.
question
Operant conditioning is part of the treatment plan to encourage speech in a child who is nearly mute. Which technique applies? a. Encourage the child to observe others talking. b. Include the child in small group activities. c. Give the child a small treat for speaking. d. Teach the child relaxation techniques.
answer
c Operant conditioning involves giving positive reinforcement for a desired behavior. Treats are rewards and reinforce speech through positive reinforcement.
question
The parent of a child with schizophrenia tearfully asks the nurse, "What could I have done differently to prevent this illness?" Select the nurse's best response. a. "Although schizophrenia results from impaired family relationships, try not to feel guilty. No one can predict how a child will respond to parental guidance." b. "Schizophrenia is a biological illness resulting from changes in how the brain and nervous system function. You are not to blame for your child's illness." c. "There is still hope. Changing your parenting style can help your child learn to cope effectively with the environment." d. "Most mental illnesses result from genetic inheritance. Your genes are more at fault than your parenting."
answer
b The parent's comment suggests feelings of guilt or inadequacy. The nurse's response should address these feelings as well as provide information. Patients and families need reassurance that the major mental disorders are biological in origin and are not the "fault" of parents. One distracter places the burden of having faulty genes on the shoulders of the parents. The other distracters are neither wholly accurate nor reassuring.
question
A nurse influenced by Peplau's interpersonal theory works with an anxious, withdrawn patient. Interventions should focus on: a. rewarding desired behaviors. b. changing the patient's self-concept. c. administering medications to relieve anxiety. d. enhancing the patient's interactions with others.
answer
d The nurse-patient relationship is structured to provide a model for adaptive interpersonal relationships that can be generalized to others. The distracters apply to theories of cognitive, behavioral, and biological therapy.
question
A patient had psychotherapy weekly for 5 months. The therapist used free association, dream analysis, and facilitated transference to help the patient understand conflicts and foster change. Select the term that applies to this method. a. Rational-emotive behavior therapy b. Cognitive-behavioral therapy c. Psychodynamic psychotherapy d. Operant conditioning
answer
c The situation describes psychodynamic psychotherapy. The distracters use other techniques.
question
Consider a therapist's statement: "The patient is homosexual but has kept this preference secret. Severe anxiety and depression occur when the patient anticipates family reactions to this sexual orientation." Which perspective is evident in the speaker? a. Theory of interpersonal relationships b. Classical conditioning theory c. Psychosexual theory d. Behaviorism theory
answer
a The theory of interpersonal relationships recognizes the anxiety and depression as resulting from unmet interpersonal security needs. Behaviorism and classical conditioning theories do not apply. A psychosexual formulation would focus on uncovering unconscious material that relates to the patient problem.
question
A psychotherapist works with an anxious, dependent patient. Which strategy is most consistent with psychoanalytic psychotherapy? a. Identifying the patient's strengths and assets b. Praising the patient for describing feelings of isolation c. Focusing on feelings developed by the patient toward the therapist d. Providing psychoeducation and emphasizing medication adherence
answer
c Positive or negative feelings of the patient toward the therapist indicate transference. Transference is a psychoanalytic concept that can be used to explore previously unresolved conflicts. The distracters are more related to biological therapy and supportive psychotherapy. Use of psychoeducational materials is a common "homework" assignment used in cognitive therapy.
question
A person says, "I was the only survivor in a small plane crash. Three business associates died. I got depressed and saw a counselor twice a week for 4 weeks. We talked about my feelings related to being a survivor and I'm OK now." Which type of therapy was used? a. Milieu therapy b. Psychoanalysis c. Behavior modification d. Interpersonal psychotherapy
answer
d Interpersonal psychotherapy returned the patient to his former level of functioning by helping him come to terms with the loss of friends and guilt over being a survivor. Milieu therapy refers to environmental therapy. Psychoanalysis would call for a long period of exploration of unconscious material. Behavior modification would focus on changing a behavior rather than helping the patient understand what is going on in his life.
question
A cognitive strategy the nurse could use to help a dependent patient would be: a. avoidance training. b. filling the patient's pill minder. c. interpreting the patient's dream content. d. examining the patient's fears related to being independent.
answer
d Cognitive theory suggests that thought processes are the basis of emotions and behavior. Changing faulty learning makes development of new adaptive behaviors possible. The distracters relate to psychoanalytic therapy, biological therapy, and aversion therapy.
question
A single parent who is employed full time complains of feelings of inadequacy related to work and family. The parent seeks help from a therapist who specializes in cognitive behavioral therapy. The therapist will treat the parent by: a. discussing ego states the parent experiences. b. negatively reinforcing undesirable behaviors. c. promoting assertive behavior at home and work. d. helping the parent identify and change faulty thinking.
answer
d Cognitive therapy emphasizes the importance of changing erroneous ways people think about themselves. Once faulty thinking is changed, the individual's behavior changes. The distracters describe a psychoanalytic approach and behavior modification.
question
A college student received an invitation to attend the wedding of a close friend who lives across the country. The student is afraid of flying. What type of therapy would the nurse suggest? a. Psychoanalysis b. Milieu therapy c. Systematic desensitization d. Short-term dynamic therapy
answer
c Systematic desensitization is a type of therapy aimed at extinguishing a specific behavior, such as the fear of flying. Psychoanalysis and short-term dynamic therapy seek to uncover conflicts. Milieu therapy involves environmental factors.
question
A patient would benefit from therapy in which peers as well as staff have a voice in determining patient privileges and psychoeducational topics. Which approach would be best? a. Milieu therapy b. Cognitive therapy c. Short-term dynamic therapy d. Systematic desensitization
answer
a Milieu therapy is based on the idea that all members of the environment contribute to the planning and functioning of the setting. The distracters are individual therapies that do not fit the description.
question
A patient asks, "What are neurotransmitters? The doctor said mine are imbalanced." Select the nurse's best response. a. "How do you feel about having imbalanced neurotransmitters?" b. "You must feel relieved to know that your problem has a physical basis." c. "Neurotransmitters are substances we eat daily that influence memory and mood." d. "Neurotransmitters are natural chemicals that pass messages between brain cells."
answer
d The patient asked for information, and the correct response is most accurate. Neurotransmitters are chemical substances that function as messengers in the central nervous system. They are released from the axon terminal, diffuse across the synapse, and attach to specialized receptors on the postsynaptic neuron. The distracters either do not answer the patient's question or provide untrue, misleading information.
question
The nurse wants to assess a patient with major depression for disturbances in circadian rhythms. Select the best question for this aspect of the assessment. a. "Have you ever seen or heard things that others do not?" b. "What are your worst and best times of day?" c. "How would you describe your thinking?" d. "Do you think your memory is failing?"
answer
b Mood changes throughout the day may be related to circadian rhythm disturbances. Questions about sleep pattern are also relevant to circadian rhythms. The distracters apply to assessment for illusions and hallucinations, thought processes, and memory.
question
The nurse administers a medication that potentiates the action of GABA. Which effect would be expected? a. Reduced anxiety b. Improved memory c. More organized thinking d. Fewer sensory perceptual alterations
answer
a Increased levels of GABA reduce anxiety. Acetylcholine and substance P are associated with memory enhancement. Thought disorganization is associated with dopamine. GABA is not associated with sensory perceptual alterations.
question
A nurse could anticipate that treatment for a patient with memory difficulties might include medications designed to: a. inhibit GABA. b. increase dopamine activity. c. reduce neurotensin metabolism. d. prevent destruction of acetylcholine.
answer
d Increased acetylcholine plays a role in learning and memory. Preventing destruction of acetylcholine by acetylcholinesterase would result in higher levels of acetylcholine, with the potential for improved memory. GABA affects anxiety rather than memory. Increased dopamine would cause symptoms associated with schizophrenia or mania rather than improve memory. Decreasing dopamine at receptor sites is associated with Parkinson's disease rather than improving memory.
question
The therapeutic action of neurotransmitter inhibitors that block reuptake cause: a. decreased concentration of the neurotransmitter in the central nervous system. b. increased concentration of neurotransmitter in the synaptic gap. c. destruction of receptor sites. d. limbic system stimulation.
answer
b If the reuptake of a substance is inhibited, it accumulates in the synaptic gap and its concentration increases, permitting ease of transmission of impulses across the synaptic gap. Normal transmission of impulses across synaptic gaps is consistent with normal rather than depressed mood. The other options are not associated with blocking neurotransmitter reuptake.
question
A nurse assesses that a patient has fear as well as increased heart rate and blood pressure. The nurse suspects increased activity of which neurotransmitter? a. GABA b. Histamine c. Acetylcholine. d. Norepinephrine
answer
d Norepinephrine is the neurotransmitter associated with sympathetic nervous system stimulation, preparing the individual for "fight or flight." GABA is a mediator of anxiety level. A high concentration of histamine is associated with an inflammatory response. A high concentration of acetylcholine is associated with parasympathetic nervous system stimulation.
question
A patient with bipolar disorder has an unstable mood, aggressiveness, agitation, talkativeness, and irritability. The nurse begins care planning based on the expectation that the health care provider is most likely to prescribe a medication classified as a(n): a. anticholinergic. b. mood stabilizer. c. psychostimulant. d. antidepressant.
answer
b The symptoms describe mania, which is effectively treated by mood stabilizers such as lithium and selected anticonvulsants (carbamazepine, valproic acid, and lamotrigine). Drugs from the other classifications listed are not effective in the treatment of mania.
question
Which intervention by a psychiatric nurse best applies the ethical principle of autonomy? The nurse: a. explores alternative solutions with a patient, who than makes a choice. b. suggests that two patients who were fighting be restricted to the unit. c. intervenes when a self-mutilating patient attempts to harm self. d. stays with a patient demonstrating a high level of anxiety.
answer
a Autonomy is the right to self-determination, that is, to make one's own decisions. By exploring alternatives with the patient, the patient is better equipped to make an informed, autonomous decision. The distracters demonstrate beneficence, fidelity, and justice.
question
Which action by a psychiatric nurse best supports the right of patients to be treated with dignity and respect? a. Consistently addresses patients by title and surname b. Strongly encourages a patient to participate in the unit milieu c. Discusses a patient's condition with the health care provider in the elevator d. Informs a treatment team that a patient is too drowsy to participate in care planning
answer
a A simple way of showing respect is to address the patient by title and surname rather than assume the patient would wish to be called by the first name. The distracters violate confidentiality and autonomy or exemplify beneficence and fidelity.
question
Two hospitalized patients fight when they are in the same room. During a team meeting, a nurse asserts that safety is of paramount importance, so treatment plans should call for both patients to be secluded to keep them from injuring each other. This assertion: a. reinforces the autonomy of the two patients. b. violates the civil rights of both patients. c. represents the intentional tort of battery. d. correctly places emphasis on safety.
answer
b Patients have a right to treatment in the least restrictive setting. Safety is important, but less restrictive measures should be tried first. Unnecessary seclusion may result in a charge of false imprisonment. Seclusion removes the patient's autonomy. The principle by which the nurse is motivated is beneficence, not justice. The tort represented is false imprisonment
question
In a team meeting a nurse says, "I'm concerned about whether we are behaving ethically in using restraint to prevent one patient from engaging in self-mutilating behavior while the care plan for another self-mutilating patient calls for one-on-one supervision." Which ethical principle most clearly applies to this situation? a. Beneficence b. Autonomy c. Fidelity d. Justice
answer
d The nurse is concerned about justice, that is, fair distribution of care, which includes treatment with the least restrictive methods for both patients. Beneficence means promoting the good of others. Autonomy is the right to make one's own decisions. Fidelity is the observance of loyalty and commitment to the patient.
question
Select the example of a tort. a. The plan of care for a patient is not completed within 24 hours of the patient's admission. b. A nurse gives a PRN dose of an antipsychotic drug to an agitated patient because the unit is short staffed. c. An advanced practice nurse recommends hospitalization for a patient who is dangerous to self and others. d. A patient's admission status changed from involuntary to voluntary after the patient's hallucinations subside.
answer
b A tort is a civil wrong against a person that violates his or her rights. Giving unnecessary medication for the convenience of staff controls behavior in a manner similar to secluding a patient; thus false imprisonment is a possible charge. The other options do not exemplify torts.
question
What is the legal significance of a nurse's action when a patient verbally refuses medication and the nurse gives the medication over the patient's objection? The nurse: a. has been negligent. b. committed malpractice. c. fulfilled the standard of care. d. can be charged with battery.
answer
d Battery is an intentional tort in which one individual violates the rights of another through touching without consent. Forcing a patient to take medication after the medication has been refused constitutes battery. The charge of battery can be brought against the nurse.
question
Which nursing intervention demonstrates false imprisonment? a. A confused and combative patient says, "I'm getting out of here and no one can stop me." The nurse restrains this patient without a health care provider's order and then promptly obtains an order. b. A patient has been irritating and attention seeking much of the day. Now a nurse escorts the patient down the hall saying, "Stay in your room or you'll be put in seclusion." c. An involuntarily hospitalized patient with suicidal ideation runs out of the psychiatric unit. The nurse rushes after the patient and convinces the patient to return to the unit. d. An involuntarily hospitalized patient with homicidal ideation attempts to leave the facility. A nurse calls the security team and uses established protocols to prevent the patient from leaving.
answer
b False imprisonment involves holding a competent person against his or her will. Actual force is not a requirement of false imprisonment. The individual needs only to be placed in fear of imprisonment by someone who has the ability to carry out the threat. If a patient is not competent (confused), the nurse should act with beneficence. Patients admitted involuntarily should not be allowed to leave without permission of the treatment team.
question
Which patient meets criteria for involuntary commitment for psychiatric treatment? The patient who: a. is noncompliant with the treatment regimen. b. fraudulently files for bankruptcy. c. sold and distributed illegal drugs. d. threatens to harm self and others.
answer
d Involuntary commitment protects patients who are dangerous to themselves or others and cannot care for their own basic needs. Involuntary commitment also protects other individuals in society. The behaviors described in the other options are not sufficient to require involuntary hospitalization.
question
A nurse at the mental health center prepares to administer a scheduled injection of haloperidol decanoate (Haldol depot) to a patient with schizophrenia. As the nurse swabs the site, the patient shouts, "Stop, stop. I don't want to take that medicine anymore because I hate the side effects." Select the nurse's first action. a. Assemble other staff for a show of force and proceed with the injection, using restraint if necessary. b. Stop the medication administration procedure and say to the patient, "Tell me more about the side effects you've been having." c. Proceed with the injection but explain to the patient that there are medications that will help reduce the unpleasant side effects. d. Say to the patient, "Since I've already drawn the medication in the syringe, I'm required to give it, but let's talk to the doctor about delaying next month's dose."
answer
b Patients with mental illness retain their civil rights unless there is clear, cogent, and convincing evidence of dangerousness. The patient in this situation presents no evidence of dangerousness. The nurse, an as advocate and educator, should seek more information about the patient's decision and should not force the medication.
question
Several nurses are concerned that agency policies related to restraint and seclusion practices are inadequate. Which understanding about the relationship of substandard institutional policies and individual nursing practice should guide nursing practice? a. The policies do not absolve an individual nurse of the responsibility to practice according to professional standards of nursing care. b. Agency policies are considered the legal standard by which a professional nurse must act and therefore override other standards of care. c. In an institution with substandard policies, the nurse has a responsibility to inform the supervisor and leave the premises without delay. d. Interpretation of policies by the judicial system is rendered on an individual basis and therefore cannot be predicted.
answer
a Nurses are professionally bound to uphold the American Nurses Association standards of practice regardless of lesser standards established by a health care agency or a state. Conversely, if the agency standards are higher than the American Nurses Association standards of practice, the agency standards must be upheld. The courts may seek to establish the standard of care through the use of expert witnesses when the issue is clouded.
question
A newly admitted acutely psychotic patient is a private patient of the chief of staff and a private-pay patient. To whom does the psychiatric nurse assigned to the patient owe the duty of care? a. Health care provider b. Hospital c. Profession d. Patient
answer
d Although the nurse is accountable to the health care provider, the agency, the patient, and the profession, the duty of care is owed to the patient.
question
Which action by a nurse shows a breach of a patient's right to privacy? a. Documents the patient's daily behaviors during hospitalization b. Releases information to the patient's employer without consent c. Discusses the patient's history with other staff during care planning d. Asks family to share information about a patient's prehospitalization behavior
answer
b Release of information without patient authorization violates the patient's right to privacy. The other options are acceptable nursing practices.
question
An adolescent hospitalized after a violent physical outburst tells the nurse, "I'm going to kill my father, but you can't tell anyone." Select the nurse's best response. a. "You're right. Federal law requires me to keep that information private." b. "Those kinds of thoughts will make your hospitalization longer." c. "You really should share this thought with your psychiatrist." d. "I am obligated to that share information with the treatment team."
answer
d Breach of nurse-patient confidentiality does not pose a legal dilemma for nurses in these circumstances because a team approach to delivery of psychiatric care presumes communication of patient information to other staff members to develop treatment plans and outcome criteria. The patient should know that the team may have to warn the father of the risk for harm.
question
A voluntarily hospitalized patient tells the nurse, "Get me the forms for discharge. I want to leave now." Select the nurse's best response. a. "I will get the forms for you right now and bring them to your room." b. "Since you signed your consent for treatment, you may leave if you desire." c. "I will get them for you, but let's talk about your decision to leave treatment." d. "I cannot give you those forms without your health care provider's knowledge."
answer
c A voluntarily admitted patient has the right to demand and obtain release in most states. However, as a patient advocate, the nurse is responsible for weighing factors related to the patient's wishes and best interests. By asking for information, the nurse may be able to help the patient reconsider the decision. Facilitating discharge without consent is not in the patient's best interests before exploring the reason for the request.
question
The family of a patient whose insurance will not pay for continuing hospitalization considers transferring the patient to a public mental hospital. They express concern that the patient will be "never get any treatment." Which reply by the nurse would be most helpful? a. "Under the law, treatment must be provided. Hospitalization without treatment violates patients' rights." b. "All patients in public hospitals have the right to choose both a primary therapist and a primary nurse." c. "That's a justifiable concern because the right to treatment extends only to provision of food, shelter, and safety." d. "Much will depend on other patients, because the right to treatment for a psychotic patient takes precedence over the right to treatment of a patient who is stable."
answer
a The right to medical and psychiatric treatment was conferred on all patients hospitalized in public mental hospitals with the enactment of the federal Hospitalization of Mentally Ill Act in 1964.
question
Which individual with mental illness may need emergency or involuntary hospitalization for mental illness? The individual who: a. resumes using heroin while still taking naltrexone (ReVia). b. reports hearing angels playing harps during thunderstorms. c. does not keep an outpatient appointment with the mental health nurse. d. throws a heavy plate at a waiter at the direction of command hallucinations.
answer
d Throwing a heavy plate is likely to harm the waiter and is evidence of dangerousness to others. This behavior meets the criteria for emergency or involuntary hospitalization for mental illness. The behaviors in the other options evidence mental illness but not dangerousness.
question
A patient in alcohol rehabilitation reveals to the nurse, "I feel terrible guilt for sexually abusing my 6-year-old before I was admitted." Select the nurse's most important action. a. Anonymously report the abuse by phone to the local child protection agency. b. Reply, "I'm glad you feel comfortable talking to me about it." c. Respect nurse-patient relationship confidentiality. d. File a written report on agency letterhead.
answer
a Laws regarding child abuse reporting discovered by a professional during the suspected abuser's alcohol or drug treatment differ by state. Federal law supersedes state law and prohibits disclosure without a court order except in instances in which the report can be made anonymously or without identifying the abuser as a patient in an alcohol or drug treatment facility.
question
A family member of a patient with delusions of persecution asks the nurse, "Are there any circumstances under which the treatment team is justified in violating a patient's right to confidentiality?" The nurse should reply that confidentiality may be breached: a. under no circumstances. b. at the discretion of the psychiatrist. c. when questions are asked by law enforcement. d. if the patient threatens the life of another person.
answer
d The duty to warn a person whose life has been threatened by a psychiatric patient overrides the patient's right to confidentiality. The right to confidentiality is not suspended at the discretion of the therapist or for legal investigations.
question
A new antidepressant is prescribed for an elderly patient with major depression, but the dose is more than the usual geriatric dose. The nurse should: a. consult a drug reference. b. teach the patient about possible side effects and adverse effects. c. withhold the medication and confer with the health care provider. d. encourage the patient to increase oral fluids to reduce drug concentration.
answer
c The dose of antidepressants for elderly patients is often less than the usual adult dose. The nurse should withhold the medication and consult the health care provider who wrote the order. The nurse's duty is to intervene and protect the patient.
question
A patient with schizophrenia believes a local minister stirred evil spirits and threatens to bomb a local church. The psychiatrist notifies the minister. Select the answer with the correct rationale. The psychiatrist: a. released information without proper authorization. b. demonstrated the duty to warn and protect. c. violated the patient's confidentiality. d. avoided charges of malpractice.
answer
b It is the health care professional's duty to warn or notify an intended victim after a threat of harm has been made. Informing a potential victim of a threat is a legal responsibility of the health care professional. It is not a violation of confidentiality.
question
A patient with psychosis became aggressive, struck another patient, and required seclusion. Select the best documentation. a. Patient struck another patient who attempted to leave day room to go to bathroom. Seclusion necessary at 1415. Plan: Maintain seclusion for 8 hours and keep these two patients away from each other for 24 hours. b. Seclusion ordered by physician at 1415 after command hallucinations told the patient to hit another patient. Careful monitoring of patient maintained during period of seclusion. c. Seclusion ordered by MD for aggressive behavior. Begun at 1415. Maintained for 2 hours without incident. Outcome: Patient calmer and apologized for outburst. d. Patient pacing, shouting. Haloperidol 5 mg given PO at 1300. No effect by 1315. At 1415 patient yelled, "I'll punch anyone who gets near me," and struck another patient with fist. Physically placed in seclusion at 1420. Seclusion order obtained from MD at 1430.
answer
d Documentation must be specific and detail the key aspects of care. It should demonstrate implementation of the least restrictive alternative. Justification for why a patient was secluded for 8 hours should be recorded, along with interventions attempted in an effort to avoid seclusion. Documentation should include a description of behavior and verbalizations, interventions tried and their outcomes, and the name of the health care provider ordering the use of seclusion.
question
A person in the community asks, "Why aren't people with mental illness kept in state institutions anymore?" Select the nurse's best response. a. "Less restrictive settings are available now to care for individuals with mental illness." b. "There are fewer persons with mental illness, so less hospital beds are needed." c. "Most people with mental illness are still in psychiatric institutions." d. "Psychiatric institutions violated patients' rights."
answer
a The community is a less restrictive alternative than hospitals for treatment of persons with mental illness. The distracters are incorrect and part of the stigma of mental illness.
question
A patient with psychosis asks a psychiatric technician, "What's the matter with me?" The technician replies, "Your wing nuts need tightening." The nurse who overheard the exchange will need to take action based on: a. violation of the patient's right to be treated with dignity and respect. b. the nurse's obligation to report caregiver negligence. c. preventing defamation of the patient's character. d. supervisory liability.
answer
a Patients have the right to be treated with dignity and respect. Patients should never be made the object of jokes about their illness. Patient emotional abuse has been demonstrated, not negligence. This example is not clearly defamation. Patient abuse, not supervisory liability, is the issue.
question
Which documentation of a patient's behavior best demonstrates a nurse's observations? a. Isolates self from others. Frequently fell asleep during group. Vital signs stable. b. Calmer; more cooperative. Participated actively in group. No evidence of psychotic thinking. c. Appeared to hallucinate. Frequently increased volume on television, causing conflict with others. d. Wore four layers of clothing. States, "I need protection from evil bacteria trying to pierce my skin."
answer
d The documentation states specific observations of the patient's appearance and the exact statements made. The other options are vague or subjective statements and can be interpreted in different ways.
question
After leaving work, a nurse realizes documentation of administration of a PRN medication was omitted. This off-duty nurse phones the unit and tells the nurse, "Please document administration of the medication for me; I forgot to do it. My password is alpha1." The nurse receiving the call should:
answer
b Fraudulent documentation may also be grounds for discipline by the state board of nursing. Referring the matter to the charge nurse will allow observance of hospital policy while ensuring that documentation occurs. Notifying the health care provider would be unnecessary when the charge nurse can resolve the problem.
question
A nurse assesses an elderly patient who was found wandering and confused. The nurse feels sad and reflects, "She's like my grandmother...so helpless." Which term best applies to the nurse's response? a. Rapport b. Transference c. Countertransference d. Defensive coping reaction
answer
c Countertransference is the nurse's response to a patient that is based on the nurse's unconscious needs, conflicts, problems, or view of the world.
question
Which statement shows a nurse has empathy for a patient who made a suicide attempt? a. "You must have been very upset to do what you did today." b. "It makes me sad to see you going through such a difficult experience." c. "If you tell me what is troubling you, I can help you solve your problems." d. "Suicide is a drastic solution to a problem that may not be such a serious matter."
answer
a Empathy permits the nurse to see an event from the patient's perspective, understand the patient's feelings, and communicate this to the patient. The distracters focus on the nurse's feelings rather than the patient's, promote patient dependence, and belittle the patient's perspective.
question
After several therapeutic encounters with a patient who recently attempted suicide, which behavior should cause the nurse to consider the possibility of countertransference? a. The patient's reactions toward the nurse seem realistic and appropriate. b. The nurse feels very happy when the patient's mood begins to lift. c. The patient states, "Talking to you is like talking to my parents." d. The nurse develops a trusting relationship with the patient.
answer
b Strong positive or negative reactions to a patient or overidentification with the patient signals possible countertransference. Nurses must carefully monitor their own feelings and reactions to detect countertransference, then seek supervision. The distracters identify desirable outcomes or transference.
question
How should the nurse respond if a patient says, "Please don't share information about me with the other people"? a. "I cannot tell anyone about you. We can help each other by keeping it between us." b. "I won't share information with your family or friends without your permission, but I will share information with other staff." c. "It depends on what you choose to tell me. I will be glad to disclose at the end of each session what I will report to other staff." d. "Therapeutic relationships are between the nurse and the patient. It's up to you to tell others what you want them to know."
answer
b A patient has the right to know with whom the nurse will share information and that confidentiality will be protected. Although the relationship is primarily between the nurse and patient, other staff needs to know pertinent data. The relationship must be patient centered and have clear boundaries.
question
A patient who recently attempted suicide talks with the nurse about wanting to take a walk on hospital grounds. The nurse responds by telling the patient, "I will talk with the psychiatrist on your behalf." Select the accurate analysis of this interaction. a. The nurse is behaving in an overly helpful way. b. The nurse is showing positive regard for the patient. c. The nurse is modeling healthy behaviors for the patient. d. The patient has manipulated the nurse into taking this action.
answer
a Being overly helpful is a classic example of role boundary blurring. The nurse's response does not reflect the modeling of healthy behavior. There is no data to indicate the patient has manipulated the nurse.
question
Termination of a therapeutic nurse-patient relationship has been successful when the nurse: a. discusses with the patient changes that happened during the relationship and evaluates outcomes. b. gives the patient the nurse's cell phone number and permission to call after discharge. c. avoids upsetting the patient by shifting focus to other patients before the patient's discharge. d. offers to meet the patient for conversation after discharge.
answer
a Summarizing and evaluating progress help validate the experience for the patient and the nurse and facilitate closure. Termination must be discussed; avoiding discussion by spending little time with the patient promotes feelings of abandonment. Successful termination requires that the relationship be brought to closure without the possibility of dependency-producing ongoing contact.
question
Select the desirable outcome for the initial stage of a nurse-patient relationship. The patient will demonstrate behaviors that indicate: a. a greater sense of independence. b. rapport and trust with the nurse. c. resolved transference. d. self-responsibility.
answer
b Development of rapport and trust is necessary before the relationship can progress to the working phase. The distracters present outcomes associated with the working phase.
question
During which phase of the nurse-patient relationship can a nurse anticipate that patient issues will be explored and resolved? a. Pre-orientation b. Orientation c. Working d. Termination
answer
c During the working phase, the nurse strives to assist the patient in making connections among dysfunctional behaviors, thinking, and emotions and offers support while alternative coping behaviors are tried.
question
At what point in the nurse-patient relationship should a nurse first address termination? a. Preorientation phase b. Orientation phase c. Working phase d. Termination phase
answer
b The patient has a right to know the conditions of the nurse-patient relationship. If the relationship is to be time limited, the patient should be informed of the number of sessions. If it is open ended, the termination date will not be known at the outset, and the patient should know that the issue will be negotiated at a later date. Termination is usually discussed during the orientation phase. The nurse is responsible for discussing termination early in the relationship.
question
Why should a nurse introduce the matter of a contract during the first session with a new patient? Contracts: a. specify what the nurse will do for the patient. b. spell out participation and responsibilities of both parties. c. are indicative of the feeling tone established between the participants. d. are binding and prevent either party from prematurely ending the relationship.
answer
b A contract emphasizes that the nurse works with the patient rather than doing something for the patient. "Working with" is a process that suggests each party is expected to participate and share responsibility for outcomes.
question
A patient frequently asks the nurse for extra snacks, implying more willingness to talk if these items are provided. The nurse should assess this behavior as: a. typical of transference reactions. b. indicative of feelings of insecurity. c. reflecting resistance to involvement. d. testing the nurse's clinical competence.
answer
d Patients often unconsciously use testing behaviors to determine whether the nurse is able to set limits or will abandon them if they behave in an unlikable way.
question
Which remark by a patient indicates movement from orientation to the working phase of a nurse-patient relationship? a. "I don't have any problems." b. "It is so difficult for me to talk about problems." c. "I don't know how talking about things twice a week can help." d. "I want to find ways to deal with my anger without blowing up."
answer
d Thinking about a more constructive approach to dealing with anger indicates a readiness to make a behavioral change. Behavioral change is associated with the working phase of the relationship. Denial or avoidance are often seen in the orientation phase.
question
A nurse explains to the family of a mentally ill patient how the nurse-patient relationship differs from other interpersonal relationships. Which is the best explanation? "The focus: a. is on the patient. Problems are discussed by the nurse and patient, but solutions are implemented by the patient." b. shifts from nurse to patient as the relationship develops. Advice is given by both and solutions implemented." c. is creation of a partnership in which each member is concerned with growth and satisfaction of the other." d. of the relationship is socialization. Mutual needs are met and feelings are shared openly."
answer
a The distracters describe events that occur in social or intimate relationships. Only the correct answer describes elements of a therapeutic relationship.
question
A nurse wants to demonstrate genuineness with a patient diagnosed with schizophrenia. The nurse should: a. use extensive self-disclosure in patient interactions. b. encourage dependence on the nurse for support and reassurance. c. consistently make interpretive judgments about the patient's behavior. d. be aware of own feelings and use congruent communication strategies.
answer
d Genuineness is a desirable characteristic involving awareness of one's own feelings as they arise and the ability to communicate them when appropriate. The other possible options are undesirable in a therapeutic relationship.
question
A nurse caring for a withdrawn, suspicious patient recognizes development of feelings of anger toward the patient. The nurse should: a. suppress the angry feelings. b. explore the anger openly and directly with the patient. c. ask the nurse manager to assign the patient to another nurse. d. discuss the anger with a clinician during a supervisory session.
answer
d The nurse is accountable for the relationship. Objectivity is threatened by strong positive or negative feelings toward a patient. Supervision is necessary to work through countertransference feelings.
question
Which action by a nurse shows positive regard? a. Making rounds according to the daily assignment. b. Administering daily medication as prescribed. c. Examining own feelings about a patient. d. Staying with a patient who is crying.
answer
d Staying with a crying patient offers support and shows positive regard. The distracters describe tasks, not necessarily with positive regard, and the nurse's efforts to remain self-aware.
question
A nurse is talking with a patient, and 5 minutes remain in the session. This patient has been silent most of the session. Another patient interrupts and says to the nurse, "I really need to talk to you." The nurse should: a. invite the interrupting patient to join the session with the current patient. b. tell the interrupting patient, "This session is 5 more minutes, then I will talk with you." c. say to the interrupting patient, "I am not available to talk with you at the present time." d. end the unproductive session with the current patient. Spend time with the next patient.
answer
b When a specific duration for sessions has been set, the nurse must adhere to the schedule. Leaving the first patient would be abandonment and may destroy trust the patient had in the nurse. Adhering to the contract demonstrates that the nurse can be trusted and that the patient and the sessions are important. The distracters violate terms of the contract.
question
A patient says, "People should be allowed to commit suicide without interference from others." A nurse replies, "You're wrong. Nothing is bad enough to justify death." What is the best analysis of this interchange? a. The patient is correct. b. Neither person is correct. c. The statements reflect differing values. d. The nurse has responded therapeutically.
answer
c Values guide beliefs and actions. The individuals stating their positions place different values on life and autonomy. Nurses must be aware of their own values and be sensitive to the values of others.
question
Which issues should a nurse address during the first interview with a patient with a psychiatric disorder? a. Trust, congruence, attitudes, and boundaries. b. Goals, resistance, unconscious motivations, and diversion. c. Relationship parameters, the contract, confidentiality, and termination. d. Transference, countertransference, intimacy, and developing resources.
answer
c Relationship parameters, the contract, confidentiality, and termination should be considered during the orientation phase of the relationship. The distracters represent issues dealt with later.
question
A psychiatric nurse visits one particular patient before work, seeks out the patient during the shift, and spends a few minutes with the patient after going off duty. Which analysis is accurate? The nurse is: a. overinvolved. b. expressing anger. c. experiencing transference. d. looking to be rescued.
answer
a The nurse behavior described is typical of overinvolvement. There are no data to support the distracters.
question
21. Which behavior shows that a nurse values autonomy? The nurse: a. sets limits on a patient's romantic overtures toward the nurse. b. suggests one-on-one supervision for a patient who is suicidal. c. informs a patient that the spouse will not be in during visiting hours. d. discusses alternatives and helps the patient weigh the consequences.
answer
d A high level of valuing is acting on one's belief. Autonomy is evident when the nurse helps patient weigh alternatives and their consequences before the patient makes a decision. Autonomy or self-determination is not the issue in any of the other behaviors.
question
A nurse provided psychiatric home care services to a patient for 6 months, but now the patient will begin a psychosocial rehabilitation program. On the nurse's final home visit, the patient gives the nurse a gold angel pin and says, "Thank you for being my guardian angel when I needed help." Select the nurse's best response. a. "I'm happy you have made so much improvement. Thank you for the pin." b. "Our agency's policies and procedures prohibit me from accepting your gift." c. "All nurses care. It's rewarding when patients recognize how hard we work." d. "I'm glad you've made progress and that I helped, but I cannot accept the gift."
answer
d Accepting a gift creates a social rather than therapeutic relationship with the patient and blurs the boundaries of the relationship. A caring nurse will acknowledge the patient's gesture of appreciation, but this gift should not be accepted.
question
A patient says, "I'm still on restriction, but I want to attend some off-unit activities. Would you ask the doctor to change my privileges?" Select the nurse's best response. a. "I'll be glad to mention it when I see the doctor today." b. "That's a good topic for you to take up with your doctor." c. "Why are you asking me when you're able to speak for yourself?" d. "I'm glad you feel comfortable asking me to help you with that request."
answer
b Nurses should encourage patients to work at their optimal level of functioning. The nurse does not act for the patient unless necessary. Acting for a patient increases feelings of helplessness and dependency.
question
A community mental health nurse has worked with a patient for 3 years but is moving and must terminate the relationship. When a new nurse begins work with this patient, what is the starting point for the relationship? a. Begin at the orientation phase b. Continue the working relationship c. Start with informal social interactions d. Return to the emotional catharsis phase
answer
a After termination of a long-term relationship, the patient and new nurse usually have to begin at ground zero, the orientation phase, to build a new relationship. If termination is successfully completed, the orientation phase sometimes progresses quickly to the working phase. Other times, even after successful termination, the orientation phase may be prolonged.
question
The nurse tells a peer, "I feel very uncomfortable with a patient and find myself wanting to avoid both informal contacts and scheduled sessions." Without supervision, which outcome is likely? a. Growing interest and mutuality b. Mutual withdrawal c. Positive regard d. Trust
answer
b A nurse's negative preconceived ideas about a patient and negative feelings toward the patient usually result in frustration and mutual withdrawal. Supervision would cause the nurse to explore the origins of the feelings and make changes as necessary. The other options suggest positive outcomes, which rarely occur in the face of negative feelings on the part of the nurse.
question
A nurse believes, "I'm the only one who truly understands this patient. Other staff members are too critical." Which situation is evident? a. Boundary blurring b. Sexual harassment c. Positive regard d. Transference
answer
a When the role of the nurse and the role of the patient shift, boundary blurring may arise. In this situation, the nurse is becoming overinvolved with the patient as a probable result of unrecognized countertransference. When boundary issues occur, the need for supervision exists. The situation does not describe sexual harassment. Data are not present to suggest positive regard or transference.
question
As a patient with mental illness is discharged from a facility, the nurse invites the patient to the annual staff holiday party. Select the best analysis of this scenario. a. The invitation facilitates dependency on the nurse. b. The nurse's action blurs the boundaries of the therapeutic relationship. c. The invitation is therapeutic for the patient's diversional activity deficit. d. The nurse's action assists the patient's integration into community living.
answer
b The invitation creates a social rather than therapeutic relationship. It may or may not increase dependency on the nurse.
question
As a nurse discharges a patient, the patient gives the nurse a card of appreciation made in an arts and crafts group. Select the nurse's best action. a. Recognize the patient's thoughtfulness. Express appreciation and accept the card. b. Inform the patient that accepting gifts violates policies of the facility. Decline the card. c. Acknowledge the patient's transition through the termination phase, but decline the card. d. Accept the card. Invite the patient to return to participate in other arts and crafts groups.
answer
a The nurse must consider the meaning, timing, and value of the gift. In this instance, the nurse should accept the patient's expression of gratitude.
question
During the first interview, a nurse notices that a patient does not make eye contact. Which analysis is correct? a. The patient is not truthful and is demonstrating evasive behavior. b. The patient is shy. The nurse should progress slowly. c. The patient feels sad and cannot look at the nurse. d. More information is needed to draw a conclusion.
answer
d The data presented are insufficient to draw a conclusion. The nurse must continue to gather information.
question
The process whereby the client unconsciously and inappropriately transfers (displaces) onto individuals the feelings, desires, and conflicts that were initially felt toward parents or significant others
answer
Transference
question
The process whereby the therapist unconsciously and inappropriately transfers (displaces) onto individuals the feelings, desires, and conflicts that were initially felt toward parents or significant others
answer
countertransference
question
The systematic management of the socio- environment as a treatment modality for the benefit of the client
answer
Milieu Therapy
question
Re-socialization Ego development ( Self understanding) Prevention of regression Saftey, Structure, Suppourt
answer
Goals of Milieu Therapy
question
A. explores alternative solutions with a patient, who than makes a choice. B. suggests that two patients who were fighting be restricted to the unit. C. intervenes when a self-mutilating patient attempts to harm self. D. stays with a patient demonstrating a high level of anxiety.
answer
Which intervention by a psychiatric nurse best applies the ethical principle of autonomy? The nurse:
question
Consistently addresses patients by title and surname Strongly encourages a patient to participate in the unit milieu Discusses a patient's condition with the health care provider in the elevator Informs a treatment team that a patient is too drowsy to participate in care planning
answer
Which action by a psychiatric nurse best supports the right of patients to be treated with dignity and respect?
question
A) Consistently addresses patients by title and surname B) Strongly encourages a patient to participate in the unit milieu C) Discusses a patient's condition with the health care provider in the elevator D) Informs a treatment team that a patient is too drowsy to participate in care planning
answer
Which action by a psychiatric nurse best supports the right of patients to be treated with dignity and respect?
question
A. reinforces the autonomy of the two patients. B. violates the civil rights of both patients. C. represents the intentional tort of battery. D. correctly places emphasis on safety.
answer
Two hospitalized patients fight when they are in the same room. During a team meeting, a nurse asserts that safety is of paramount importance, so treatment plans should call for both patients to be secluded to keep them from injuring each other. This assertion:
question
- corrects maldaptive behavior -develops adaptive coping -develops stress management skills - improves interpersonal skills
answer
Group Therapy does what for the patient?
question
prescribe psychotropic medication
answer
a new staff nurse completes an orientation to the psychiatric unit. this nurse will expect to ask an advanced practice nurse to perform which action for patients
question
risk for suicide
answer
a newly admitted patient diagnosed with major depression has gained 20 lbs over a few months and has suicidal ideation. the patient has taken an antidepressant medication for 1 week without remission of symptoms. select the priority nursing diagnosis
question
implement suicide precautions
answer
a patient diagnosed with major depression has lost 20 lbs in one month, has chronic low self-esteem, and a plan for suicide. the patient has taken an antidepressant medication for 1 week. which nursing intervention has the highest priority?
question
Examine interventions for possible revision of the target date.
answer
The desired outcome for a patient experiencing insomnia is, "Patient will sleep for a minimum of 5 hours nightly within 7 days." At the end of 7 days, review of sleep data shows the patient sleeps an average of 4 hours nightly and takes a 2-hour afternoon nap. What is the nurse's next action?
question
Implementation
answer
A patient begins a new program to assist with building social skills. In which part of the plan of care should a nurse record the item, "Encourage patient to attend one psychoeducational group daily"?
question
assess the patient based on data collected from all sources.
answer
Before assessing a new patient, a nurse is told by another health care worker, "I know that patient. No matter how hard we work, there isn't much improvement by the time of discharge." The nurse's responsibility is to:
question
Assess the patient for a history of renal problems.
answer
A patient presents to the emergency department with mixed psychiatric symptoms. The admission nurse suspects the symptoms may be the result of a medical problem. Lab results show elevated BUN (blood urea nitrogen) and creatinine. What is the nurse's next best action?
question
Suicide precautions
answer
A patient states, "I'm not worth anything. I have negative thoughts about myself. I feel anxious and shaky all the time. Sometimes I feel so sad that I want to go to sleep and never wake up." Which nursing intervention should have the highest priority?
question
select and participate in one group activity per day.
answer
Select the best outcome for a patient with the nursing diagnosis: Impaired social interaction related to sociocultural dissonance as evidenced by stating, "Although I'd like to, I don't join in because I don't speak the language very well." Patient will:
question
carrying out interventions and coordinating care.
answer
Nursing behaviors associated with the implementation phase of nursing process are concerned with:
question
"I hear evil voices that tell me to do bad things."
answer
Which statement made by a patient during an initial assessment interview should serve as the priority focus for the plan of care?
question
"S: States, 'I feel like I'm ready to blow up.' O: Pacing hall, mumbling to self. A: Auditory hallucinations. P: Offer haloperidol (Haldol) 2 mg po. I: Haloperidol (Haldol) 2 mg po given at 0900. E: Returned to lounge at 0930 and quietly watched TV."
answer
Which entry in the medical record best meets the requirement for problem-oriented charting?
question
Obtain important information from the family member.
answer
A nurse assesses an older adult patient brought to the emergency department by a family member. The patient was wandering outside saying, "I can't find my way home." The patient is confused and unable to answer questions. Select the nurse's best action.
question
Cognition
answer
A nurse asks a patient, "If you had fever and vomiting for 3 days, what would you do?" Which aspect of the mental status examination is the nurse assessing?
question
"What you say about feelings is private, but some things, like suicidal thinking, must be reported to the treatment team."
answer
An adolescent asks a nurse conducting an assessment interview, "Why should I tell you anything? You'll just tell my parents whatever you find out." Which response by the nurse is appropriate?
question
"What did you have for breakfast this morning?"
answer
A nurse wants to assess an adult patient's recent memory. Which question would best yield the desired information?
question
"Are you having difficulty hearing when I speak?"
answer
When a nurse assesses an older adult patient, answers seem vague or unrelated to the questions. The patient also leans forward and frowns, listening intently to the nurse. An appropriate question for the nurse to ask would be:
question
coping strategies
answer
At what point in an assessment interview would a nurse ask, "How does your faith help you in stressful situations?" During the assessment of:
question
milieu management.
answer
When a new patient is hospitalized, a nurse takes the patient on a tour, explains rules of the unit, and discusses the daily schedule. The nurse is engaged in:
question
Determining the goals and outcome criteria
answer
After formulating the nursing diagnoses for a new patient, what is a nurse's next action?
question
Social isolation
answer
Select the most appropriate label to complete this nursing diagnosis: ___________ related to feelings of shyness and poorly developed social skills as evidenced by watching television alone at home every evening.
question
Quality and Safety Education for Nurses
answer
"QSEN" refers to:
question
Impaired verbal communication
answer
A nurse documents: "Patient is mute despite repeated efforts to elicit speech. Makes no eye contact. Inattentive to staff. Gazes off to the side or looks upward rather than at speaker." Which nursing diagnosis should be considered?
question
"Are you comfortable conversing in English, or would you prefer to have a translator present?"
answer
A nurse prepares to assess a new patient who moved to the United States from Central America three years ago. After introductions, what is the nurse's next comment?
question
meets standards
answer
The nurse records this entry in a patient's progress notes: Patient escorted to unit by ER nurse at 2130. Patient's clothing was dirty. In interview room, patient sat with hands over face, sobbing softly. Did not acknowledge nurse or reply to questions. After several minutes, abruptly arose, ran to window, and pounded. Shouted repeatedly, "Let me out of here." Verbal intervention unsuccessful. Order for stat dose 2 mg haloperidol PO obtained; medication administered at 2150. By 2215, patient stopped shouting and returned to sit wordlessly in chair. Patient placed on one-to-one observation. How should this documentation be evaluated?
Get an explanation on any task
Get unstuck with the help of our AI assistant in seconds
New